LSAT and Law School Admissions Forum

Get expert LSAT preparation and law school admissions advice from PowerScore Test Preparation.

User avatar
 Dave Killoran
PowerScore Staff
  • PowerScore Staff
  • Posts: 5852
  • Joined: Mar 25, 2011
|
#45455
Complete Question Explanation
(The complete setup for this game can be found here: lsat/viewtopic.php?t=13934)

The correct answer choice is (E)

If Walter has exactly two of the three symptoms, then he cannot have illness M (which is characterized by all three symptoms). Unfortunately, M does not appear in any of the answer choices. However, even though M does not appear in this question, it is always worth taking a chance to quickly check the answers for the presence of such a “question solving” variable.

The next step is to check pairs of variables that would, in combination, exhibit all three symptoms (you could also seek a pair of illnesses that conceivably exhibit just one symptom, but L and N—the only such pair—do not have to have the same symptom). There are two pairs that must exhibit all three symptoms when combined: J and K, and L and N (each pair contains three symptoms, but none of the symptoms can be the same). J and K do not appear in the answer choice, so let’s examine L and N, which appear in answer choice (E).

L and N together exhibit all three symptoms, since L has two symptoms and N has one symptom, and from the fifth rule they cannot have any symptoms in common. So, if Walter had L and N, he would have all three symptoms (although we cannot determine exactly which symptoms come from L and exactly which symptoms come from N). Because Walter has exactly, two symptoms, he cannot have both L and N, and answer choice (E) is correct.
 salty
  • Posts: 8
  • Joined: Jun 25, 2014
|
#17323
Hi,

My final diagram for this question set is as below:

N L L
M M M
K J J
_ _ _
F H S

F,H,S - symptoms
N,L,M,K,J - illness

With this said, I narrowed down the answer choices to (C) and (E) and I cannot figure out
the answer between the two and obviously I missed out something from the diagram, but I cannot track where I missed out. Can someone please help? Thanks.
 BethRibet
PowerScore Staff
  • PowerScore Staff
  • Posts: 200
  • Joined: Oct 17, 2012
|
#17333
Hi Salty,

The rules and inferences we draw do tell us that:

J = H + S
K = F (from rule 2)
L = H/S or both rule 3), and L has two symptoms (rule 4 & 5)
N = one symptom (rule 5) and doesn't overlap with L
and M = all three.

Your diagram fixes the symptoms for L as H+S, and then N as F. If that were true, then L & K could not overlap. But there's nothing in the rules that tells us for sure that L must have H + S, only that L has two symptoms, and at least one of them is H OR S. Given that, it's possible that L does come with a fever, and therefore, that it has a symptom in common with K. That's how we rule out answer choice C.

Hope that helps!

Beth
 awalker71
  • Posts: 2
  • Joined: Aug 01, 2018
|
#49538
Hello,

I've evaluated this problem and fully understand the reasons behind (C) being the correct answer choice. However, like L, J also has the symptoms H&S. I'm not understanding why (B) would be incorrect in this scenario. Thanks so much.
 Rachael Wilkenfeld
PowerScore Staff
  • PowerScore Staff
  • Posts: 1358
  • Joined: Dec 15, 2011
|
#49623
Hi Awalker,

This is a really tricky question. In fact, both answer choices B and C are incorrect, and for essentially the same reason. It all comes down to the question type here. The question is asking for what cannot be true. That means that under no circumstances can the answer choice be correct. Let's take a look at each one of B, C, and E (the correct choice).

For B, it says J and N. In order for the answer to be correct, it must involve either 3 symptoms or 1 symptom only. J already involves 2 symptoms (S and H) so for B to be the correct answer, N would have to have F. But does N have to have F? We know N does not share any symptoms with L. But L could have S and F (or H and F) putting either S/H on N. That would leave a situation where J has S and H and N has either S or H, for a total of 2 symptoms. This shows that B isn't the right answer, because it doesn't force there to be 3 or 1 symptoms.

For C it says K and L. For K we know it's only F. For L we have either S/H, but again, like in B, the second slot of L is open. It could be F as in K, and then there would only be 2 total symptoms (F and S/H). So C doesn't force us to have 3 symptoms or 1 symptom.

For E it says L and N. By rule these have to be different symptoms (L and N have no symptoms in common). Since L has two symptoms and N has 1 symptom, this would be three total symptoms. This would be our correct answer to which 2 illnesses cannot occur together if there are only 2 symptoms total.

Hope that helped.
Rachael
 momgoingbacktoschool
  • Posts: 65
  • Joined: Aug 11, 2020
|
#79531
Sorry that this is unnecessary, but anyone else think to themselves "WHO'S WALTER?!" and look at the stimulus again frantically thinking you missed that there was a Walter?

Get the most out of your LSAT Prep Plus subscription.

Analyze and track your performance with our Testing and Analytics Package.